LSAT and Law School Admissions Forum

Get expert LSAT preparation and law school admissions advice from PowerScore Test Preparation.

 Administrator
PowerScore Staff
  • PowerScore Staff
  • Posts: 8916
  • Joined: Feb 02, 2011
|
#24060
Complete Question Explanation

Weaken. The correct answer choice is (B)

This is a very straightforward stimulus and its argument is easy to follow. We are told that since investor owned hospitals require less public investment, have fewer employees and have higher occupancy rates they are better at delivering medical care than nonprofit hospitals of the same size. This is a Weaken question and so we will try to demonstrate that even though these premises may be true, the stimulus’s conclusion, investor owned hospitals are better than nonprofit ones, does not necessarily follow.

Answer Choice (A): This could actually strengthen the conclusion in the stimulus. If nonprofit hospitals charge more per bed, it would seem that more people could be better served by investor owned hospitals.

Answer Choice (B): This is the correct answer choice. It is to the significant advantage of nonprofit hospitals if their patients recover more quickly than those at investor owned hospitals. And so if this were true, the conclusion that investor owned hospitals are a better way of delivering medical care is severely weakened.

Answer Choice (C): This could be true but it does nothing to weaken the conclusion. Spending more time with fundraisers does not affect the fact that investor owned hospitals deliver better medical care.

Answer Choice (D): This is a tempting answer choice since we might assume that a higher paid doctor is obviously a better doctor. But we do not know if this is actually the case and so we cannot make that assumption. Because of that this answer choice does not necessarily weaken the conclusion.

Answer Choice (E): This answer choice is somewhat similar to (D) in that we might be tempted to assume that more money (in the form of donations) means better medical care. But once again we do not know if that is true, and so we cannot say that this definitely weakens the conclusion.

Get the most out of your LSAT Prep Plus subscription.

Analyze and track your performance with our Testing and Analytics Package.